In the diagram below, quadrilateral DEFG is inscribed in circle H. Solve for x and y.

In The Diagram Below, Quadrilateral DEFG Is Inscribed In Circle H. Solve For X And Y.

Answers

Answer 1

Answer:

x = 100

y = 44

Step-by-step explanation:

The interior angles of a quadrilateral add up to 360 degrees, so we know that when all four angles are added together, they need to equal 360.

121 + 111 + x - 31 + 2y - 29 = 360

Additionally, the angles opposite each other equals 180 degrees. This means D + F = 180 degrees, and E + G = 180 degrees.

111 + (x - 31) = 180

Subtract 111 from both sides.

x - 31 = 69

Add 31 to each side

x = 100

Plug in the value for x to check the answer

111 + (100 - 31) = 180

Then, for y, we have the same set up

121 + (2y - 29) = 180

Subtract 121 from both sides

2y - 29 = 59

Add 29 to each side

2y = 88

Divide each side by 2

y = 44

Plug in the value for y to check the answer

121 + (2*44 - 29) = 180

This means that angle F equals 59 degrees and angle G equals 69 degrees.

x = 100

y = 44


Related Questions

The diagram shows the net of a square pyramid and its dimensions. A net of a square pyramid. The square base has a base and height of 6 feet. There are four triangles that have a base of 6 feet and a height of 4 feet. What is the total surface area of the pyramid in square feet? Record your answer in the box.

Answers

A net of a square pyramid. The square base has a base and height of 6 feet. There are four triangles that have a base of 6 feet and a height of 4 feet.

To find the total surface area of the pyramid, we need to find the area of each of the five faces and then add them up.

The area of the square base is

6 ft x 6 ft = 36 sq ft

Each of the four triangular faces has an area of

(1/2) x base x height = (1/2) x 6 ft x 4 ft = 12 sq ft

So the total area of the four triangular faces is

4 x 12 sq ft = 48 sq ft

Finally, we need to find the area of the fifth face, which is the bottom of the pyramid. Since it is also a square with side length 6 ft, its area is

6 ft x 6 ft = 36 sq ft

Therefore, the total surface area of the pyramid is

36 sq ft (square base) + 48 sq ft (four triangular faces) + 36 sq ft (bottom) = 120 sq ft.

 

[tex]\left[\begin{array}{ccc}\1120 square feet\\\end{array}\right][/tex]

Therefore, the total surface area of the pyramid is 120 square feet.

To know more about square pyramid here

https://brainly.com/question/27681171

#SPJ1

an insurance policy reimburses dental expense, , up to a maximum benefit of 250. the probability density function for is where is a nonzero constant. calculate the median benefit for this policy.

Answers

The expression represents the median benefit for this insurance policy is x = [250 - √(62500 + 4/c)] / 2

To determine the median benefit for this policy, we need to first understand the probability density function (PDF) associated with the policy. The PDF is a mathematical function that describes the probability of a random variable taking on a specific value. In this case, the random variable is the benefit amount provided by the insurance policy.

The cumulative distribution function (CDF) for this PDF is F(x) = cx(250 - x)/2. To find the value of x that corresponds to a cumulative probability of 0.5, we set F(x) = 0.5 and solve for x:

0.5 = cx(250 - x)/2

1 = cx(250 - x)

1/c = x(250 - x)

1/c = -x² + 250x

0 = -x² + 250x - 1/c

We can solve this quadratic equation using the quadratic formula, which gives us:

x = [250 ± √(250² - 4(-1/c)(-1))] / 2

Simplifying this expression gives us:

x = [250 ± √(62500 + 4/c)] / 2

Since we know that x cannot exceed 250, we take the negative root of the expression to get:

x = [250 - √(62500 + 4/c)] / 2

To know more about probability here

https://brainly.com/question/11234923

#SPJ4

solve for z in the following equation $1 iz 1 iz where i 2 1 simplify your answer as much as possible

Answers

The solution for z is 0.

What is the equivalent expression?

Equivalent expressions are expressions that perform the same function despite their appearance. If two algebraic expressions are equivalent, they have the same value when we use the same variable value.

Assuming the equation is:

1 + iz = 1 - iz

We can start by isolating the term with z on one side:

1 + iz = 1 - iz

2iz = 0

Divide both sides by 2i:

z = 0

Therefore, the solution for z is 0.

To learn more about the equivalent expression visit:

https://brainly.com/question/2972832

#SPJ4

Complete Question:

Solve for z in the following equation: 1-iz = -1 + iz (where i^2 = -1).

Simplify your answer as much as possible.

In triangle ABC, BG = 24 mm. What is the length of segment GE?

Answers

The length of segment GE is 6√15 mm.

To find the length of segment GE, we need to use the fact that the medians of a triangle are concurrent at a point called the centroid, which divides each median into two segments in a 2:1 ratio. Specifically, the segment of the median that connects the centroid to a vertex is twice as long as the segment that connects the centroid to the midpoint of the opposite side.

Let M be the midpoint of BC, and let GE intersect AM at point H. Then, we know that GH is twice as long as HM, and we also know that GM is one-third the length of AM. Therefore, we can write:

GH = 2HM

GM = (1/3)AM

We can also use the fact that the medians of a triangle divide each other into segments in a 2:1 ratio. Specifically, we know that BD = (2/3)BM and CD = (2/3)CM. Since BG is a median, we know that BG = (2/3)BD, so we can write:

BG = (2/3)BD

24 mm = (2/3)(2/3)BM

BM = 27 mm

Now we can use the fact that GM is one-third the length of AM to find AM:

GM + MH = AM

(1/3)AM + GH = AM

GH = (2/3)AM

Substituting the expressions we found for GH and BM into the above equation, we get:

2HM = (2/3)AM - (1/3)AM

2HM = (1/3)AM

HM = (1/6)AM

We also know that BM = CM, since M is the midpoint of BC. Therefore, we can write:

BC = BM + CM

BC = 2BM

BC = 54 mm

Using the Pythagorean theorem, we can find AM:

[tex](AM)^{2}[/tex] = [tex](AG)^{2}[/tex] - [tex](GM)^{2}[/tex]

[tex](AM)^{2}[/tex]  = [tex](2BG)^{2}[/tex] - (1/9)[tex](BC)^{2}[/tex]

[tex](AM)^{2}[/tex] = 4[tex](24)^{2}[/tex] - (1/9)[tex](54)^{2}[/tex]^2

[tex](AM)^{2}[/tex] = 576 - 324/9

[tex](AM)^{2}[/tex] = 576 - 36

[tex](AM)^{2}[/tex] = 540

AM = √540 mm

AM = 6√60 mm

Finally, we can find GE by using the fact that GH is twice as long as HM:

GH = 2HM

GH = 2(1/6)AM

GH = (1/3)AM

Therefore, we can write:

GE = GH + HE

GE = (1/3)AM + (1/2)HM

GE = (1/3)(6√60 mm) + (1/2)(1/6)(6√60 mm)

GE = 2√60 mm + √60 mm

GE = 3√60 mm

Simplifying, we get:

GE = 3√60 mm = 6√15 mm

Correct Question :

In triangle ABC, BG = 24 mm. What is the length of segment GE where G is the point where all the medians meet and D, E and F are the points on the sides where median meet the side.

To learn more about length here:

https://brainly.com/question/22681923

#SPJ1

Use the standard normal distribution or theâ t-distribution to construct a 95â% confidence interval for the population mean. Justify your decision. If neither distribution can beâ used, explain why. Interpret the results.
In a recentâ season, the population standard deviation of the yards per carry for all running backs was 1.35. The yards per carry of 25 randomly selected running backs are shown below. Assume the yards per carry are normally distributed. 1.5

Answers

The true population mean of yards per carry for all running backs, according to our 95% confidence level, is between 3.52 and 4.592 yards per carry.

We may use the Z-distribution to create a 95% confidence interval for the population mean because the population standard deviation is known and the sample size is more than 30.

The formula for the confidence interval is:

[tex]CI = x + Z\alpha /2 * \alpha /\sqrt{n}[/tex]

Where is the population standard deviation, n is the sample size, x is the sample mean, Z/2 is the Z-score corresponding to the desired degree of confidence (in this example, 95%), and is the Z-score.

From the given data, we have:

Sample mean [tex](x) = 4.056[/tex]

Population standard deviation [tex](\alpha ) = 1.35[/tex]

Sample size [tex](n) = 25[/tex]

Using a Z-table or calculator, we can find that the Z-score for a 95% confidence level is 1.96 (rounded to two decimal places).

When the values are added to the formula, we obtain:

[tex]CI = 4.056 + 1.96 * 1.35/x^{25}[/tex]

[tex]CI = 4.056 + 0.536[/tex]

[tex]CI = (3.52, 4.592)[/tex]

To know more about Normal Distribution visit:

https://brainly.com/question/29509087

#SPJ4

a car was driven 18 different times with different octane levels. using the output from the regression, give a 71% confidence interval for the effect of octane on the car. use 3 decimal places. simple linear regression results: dependent variable: mileage

Answers

Based on the simple linear regression results for the car's mileage, we can estimate the effect of octane levels on the car's performance.

Using a 71% confidence interval and rounding to 3 decimal places, we can say with some confidence that the impact of octane levels on the car's mileage is between -0.045 and 0.102.

This means that while there is some correlation between octane levels and mileage, the effect is relatively small and falls within a narrow range of values. It's worth noting that there may be other factors at play that influence the car's performance, so this result should be interpreted with caution.

Learn more about linear regression here:

brainly.com/question/14313391

#SPJ11

the area of the largest equilateral triangle that can be inscribed in a square of side length unit can be expressed in the form square units, where and are integers. what is the value of ?

Answers

The area of the largest equilateral triangle that can be inscribed in a square of side length 1 unit is (1/4) * √3 square units.



To find the area of the largest equilateral triangle that can be inscribed in a square of side length 1 unit, follow these steps:

1. Draw an equilateral triangle inside the square with one of its vertices touching the midpoint of the bottom side of the square, and the other two vertices touching the midpoints of the other two sides.

2. The height (h) of the equilateral triangle can be found using Pythagorean theorem. Since the triangle is equilateral, it can be split into two 30-60-90 right triangles. In this case, the shorter leg (a) is half the side length of the square (1/2), and the longer leg (b) is the height of the equilateral triangle (h).

3. In a 30-60-90 triangle, the ratio of the sides is a:b:h = 1:√3:2. Therefore, we can write the equation:

1/2 : h : 1

4. To find the value of h, we can set up the proportion:

(1/2) / h = 1 / √3

5. Cross-multiply to solve for h:

h = (1/2) * √3

6. Now we can find the area (A) of the equilateral triangle using the formula:

A = (1/2) * base * height

In this case, the base is the side length of the square (1 unit) and the height is h:

A = (1/2) * 1 * ((1/2) * √3)

7. Simplify the expression:

A = (1/4) * √3 square units

So, the area of the largest equilateral triangle that can be inscribed in a square of side length 1 unit is (1/4) * √3 square units.

to learn more about equilateral triangle click here:

brainly.com/question/2456591

#SPJ11

1) use mathematical induction to show that if you draw n straight lines in the plane you only need two colors to color the regions formed so that no two regions that have an edge in common have a common color.

Answers

By the principle of mathematical induction, we have proved that P(n) is true for all positive integers n, which completes the proof.

Let P(n) be the statement "If n straight lines are drawn in the plane, the regions formed can be colored with two colors in such a way that no two regions that share a common edge have the same color."

Base case: P(1) is true, as there is only one line and it divides the plane into two regions that can be colored with two different colors.

Inductive hypothesis: Assume P(k) is true for some arbitrary value of k, where k is a positive integer. That is, we assume that if k straight lines are drawn in the plane, the regions formed can be colored with two colors in such a way that no two regions that share a common edge have the same color.

Inductive step: We want to prove that P(k+1) is true, i.e., if (k+1) straight lines are drawn in the plane, the regions formed can be colored with two colors in such a way that no two regions that share a common edge have the same color.

Consider the (k+1)th line. It intersects the existing k lines in k+1 points, dividing the plane into k+2 regions. We can choose one of these regions to be unbounded and assign it one color, say white. We can then color the remaining k+1 regions with two colors (white and black) such that no two regions that share a common edge have the same color, by using the coloring scheme for P(k) that we assumed to be true.

Now we have colored k+2 regions in total, with no two adjacent regions sharing the same color. Therefore, P(k+1) is true.

Hence, By the principle of mathematical induction, we have proved that P(n) is true for all positive integers n, which completes the proof.

To know more about regions check the below link:

https://brainly.com/question/28747454

#SPJ4

Complete questions:

Use mathematical induction to show that if you draw lines in the plane you only need two colors to color the regions formed so that no two regions that have an edge in common have a common color.

The sides of rhombus are 17cm long. If one diagonal is 30cm long, what is the length of the other diagonal?​

Answers

Answer:

Step-by-step explanation:

The length of the other diagonal is 26cm. This can be calculated using the formula for a rhombus, which states that the product of the diagonals is equal to the square of the length of the sides. Therefore, 30cm * 26cm = 17cm * 17cm, so the other diagonal is 26cm.

PLEASE HELP ME . I've been asking the same question many times write a ratio in fraction from and in lowest terms for each of the following □∆∆□□∆oo∆

a) squares to triangles

b) circles to squares to triangles

c) triangles to circles and squares​

Answers

the answer is b because the ratio is squares, triangles, circles

Answer:

1.3/4

2.2/3/4 or 1/6

3.4/2/3 or 2/3

evaluate each integral by interpreting it in terms of areas. (a) 8 0 f(x) dx (b) 20 0 f(x) dx (c) 28 20 f(x) dx (d) 28 12 f(x) dx (e) 28 12 |f(x)| dx (f) 0 8 f(x) dx

Answers

To evaluate each integral in terms of areas, we need to understand that the integral represents the area under the curve of a function, f(x), between two points on the x-axis.

Let's discuss each integral:

(a) ∫₀⁸ f(x) dx: This represents the area under the curve of f(x) from x = 0 to x = 8. The integral calculates the accumulated area along this interval.

(b) ∫₀²⁰ f(x) dx: Similarly, this represents the area under the curve of f(x) from x = 0 to x = 20. It's a broader interval than (a), so it covers more area under the curve.

(c) ∫²⁰²⁸ f(x) dx: This integral represents the area under the curve of f(x) between x = 20 and x = 28. It's important to note that the interval is now shifted to the right compared to (a) and (b).

(d) ∫¹²²⁸ f(x) dx: This integral calculates the area under the curve of f(x) from x = 12 to x = 28. The interval here is larger than in (c), covering more area under the curve.

(e) ∫¹²²⁸ |f(x)| dx: This integral evaluates the area under the absolute value of f(x) from x = 12 to x = 28. The absolute value ensures that negative function values contribute positively to the area calculation, preventing any cancelation of areas.

(f) ∫₀⁸ f(x) dx: This integral is the same as (a), representing the area under the curve of f(x) from x = 0 to x = 8.

Each integral evaluates the area under the curve of f(x) for different intervals on the x-axis, providing insights into the total accumulated area in those intervals.

To learn more about function  click here

brainly.com/question/12431044

#SPJ11

Which two (2) details from the text BEST

support your answer to Question 4? From the eyes have it

Answers

Blindness and eyesight are two different physical conditions that affect an individual's ability to see.

Even with good eyesight, there are limitations to our perception that can cause us to miss important details or fail to see what is right in front of us.

The narrator feels that people with good eyesight fail to see what is right in front of them because of the limitations of their perception. Our eyesight only allows us to see a limited portion of the electromagnetic spectrum, which means that there are many things in the world that we cannot see.

In mathematical terms, we can think of eyesight as a function that maps the input of light waves to the output of an image in our brain. However, this function has limitations in terms of the range of inputs it can handle and the accuracy of the output it produces.

This means that even with good eyesight, there are certain inputs that our eyes cannot handle and certain details that our brain cannot fully process.

To know more about eye sight here

https://brainly.com/question/14893189

#SPJ4

Complete Question:

How are blind people different from people with eyesight? Why does the narrator feel that people with good eye sight fail to see what is right in front of them?

HELP ME PRETTY PLEASE IM STRUGGLING

Answers

The measure of x in the parallel line is 57 degrees.

How to find the angles in a parallel line?

When parallel lines are crossed by a transversal line, angle relationships

are formed such as alternate interior angles, alternate exterior angles,

same side interior angles, vertically opposite angles, corresponding angles

etc.

Therefore, let's use the angle relationship to find the angle x as follows:

x + 123 = 180(same side interior angles)

subtract 123 from both sides of the equation

x = 180 - 123

Therefore,

x = 57 degrees

learn more on angles here: https://brainly.com/question/29430493

#SPJ1

Which of the following must be true for some c in the interval (0,10) ? f′(c)=11−(−4)/10−0 since the Mean Value Theorem applies.

Answers

For a continuous function, f and f(0) = -4 and f(10)= 1, the true value for some c in the interval (0,10) is equals the [tex]f'(c) = \frac{ 11 - (-4) }{10 - 0}[/tex] since mean value theorem applies.So, option(c) is right one.

The Mean Value Theorem is an important for determining the maximum and minimum values of a function on an interval. It is states that if a function is continuous on the interval [a, b] and differentiable on the open interval (a, b), then, there exists at least one point c∈(a,b) such that [tex]f'(c) = \frac{ f(b) - f(a)}{b - a}[/tex]. We have a function f is differentiable with f(0) = -4 and f(10) = 11.

We have to determine the true value for some c in the interval (0,10). According to mean value theorem, [tex]f'(c) = \frac{ f(b) - f(a)}{b - a}[/tex]

here, a = 0, b = 10 and f(0) = -4, f(10) = 11 so, we can write as [tex]f'(c) = \frac{ 11 -(-4) }{10 - 0}[/tex]

[tex]= \frac{ 11 + 4}{10 }[/tex] = 1.5

which is equivalent to expression present in option(c) in above figure. Hence, right option is option (c).

For more information about mean value theorem, visit :

https://brainly.com/question/30409016

#SPJ4

Complete question:

The above figure complete the question.

As the error bound of the proportion (EBP) increases, what is the effect on the sample size?

Answers

There is an inverse relationship between the EBP and the required sample size. As the EBP increases, the required sample size decreases, and vice versa.

As the error bound of the proportion (EBP) increases, the required sample size decreases.

The EBP is a measure of the maximum error that is allowed in estimating a population proportion using a sample proportion. It is calculated as the difference between the sample proportion and the true population proportion, divided by the sample proportion. For example, an EBP of 0.02 means that the estimated proportion could differ from the true population proportion by up to 2%.

When the EBP is large, it means that the allowable margin of error is also large, so we do not need as large a sample size to achieve the desired level of precision in our estimate. In other words, if we are willing to tolerate a larger error in our estimate, we can use a smaller sample size to achieve the same level of accuracy.

Conversely, when the EBP is small, it means that the allowable margin of error is also small, so we need a larger sample size to achieve the desired level of precision in our estimate. This is because a smaller margin of error requires a more precise estimate, which in turn requires a larger sample size to reduce the effect of random sampling variability.

Therefore, there is an inverse relationship between the EBP and the required sample size. As the EBP increases, the required sample size decreases, and vice versa.

Learn more about inverse relationship

https://brainly.com/question/28628342

#SPJ4

what is the length of the common tangent between two non-intersecting circles with radii r and 3r, having the shortest distance between them equal to the radius of the first circle?

Answers

the length of the common tangent between the two circles is [tex](sqrt(37)/2)r.[/tex]

What is Pythagoras theorem?

The hypotenuse's square is equal to the sum of the squares of the other two sides if a triangle has a straight angle (90 degrees), according to the Pythagoras theorem. Keep in mind that BC2 = AB2 + AC2 in the triangle ABC signifies this. Base AB, height AC, and hypotenuse BC are all used in this equation. The longest side of a right-angled triangle is its hypotenuse, it should be emphasized.

The sum of the radii of the two circles is 4r. We can draw a right triangle with legs of length d/2 and (4r - r) = 3r, and with a hypotenuse equal to the length of the common tangent.

Using the Pythagorean theorem, we can find the length of the hypotenuse:

[tex]h^2 = (d/2)^2 + (3r)^2h^2 = (d^2)/4 + 9r^2[/tex]

Multiplying both sides by 4:

[tex]4h^2 = d^2 + 36r^2[/tex]

Since d = r, we can substitute:

[tex]4h^2 = r^2 + 36r^24h^2 = 37r^2[/tex]

Dividing both sides by 4:

h^2 = (37/4)r^2

Taking the square root of both sides:

[tex]h = (sqrt(37)/2)r[/tex]

Therefore, the length of the common tangent between the two circles is [tex](sqrt(37)/2)r.[/tex]

Learn more about Pythagoras theorem, by the following link

https://brainly.com/question/231802

#SPJ4

2x=-x+3

Will mar you brainlist

Answers

X=1 Because you move the -x to the left side therefore you add it to both sides and get 3x=3 if you divide both sides by 3 you get 1. So X=1

After deducting grants based on need, the average cost to attend the University of Southern California (USC) is $27,175. Assume the population standard deviation is $7,400. Suppose that a random sample of 69 USC students will be taken from this population.
(a)
What is the value of the standard error of the mean? (Round your answer to the nearest whole number.)
$
(b)
What is the probability that the sample mean will be more than $27,175?
(c)
What is the probability that the sample mean will be within $1,000 of the population mean? (Round your answer to four decimal places.)
(d)
What is the probability that the sample mean will be within $1,000 of the population mean if the sample size were increased to 100? (Round your answer to four decimal places.)

Answers

a) The standard error of the mean value is 890.

b) 0.5 is the probability that the sample mean will be more than $27,175.

c)  [tex]11%[/tex] of the population means being within [tex]$1,000[/tex] of the sample mean.

d) The population mean is [tex]71%.[/tex] .

(a) The formula for calculating the standard error of the mean (SE) is as follows:[tex]SE = / sq rt(n),[/tex] where n is the sample size and is the population standard-deviation.

Inputting the values provided yields:

[tex]SE = 7,400 sq/69 890[/tex]

The standard error of the mean, rounded to the closest whole number, is [tex]890.[/tex]

[tex]= 890[/tex]

(b) We must standardize the sample mean using the following method in order to determine the likelihood that the sample mean will exceed [tex]$27,175:[/tex]

z is equal to[tex](x - ) / ( / sort(n)).[/tex]

where n is the sample size, x is the sample mean, is the population standard deviation, and is the population mean (which is assumed to be equal to the sample mean because it is not provided).

We obtain the following by substituting the above values: [tex]z = (27,175 - 27,175) / (7,400 / sqrt(69)) = 0.[/tex]

Obtaining a z-score of [tex]0[/tex] or above has a [tex]0.5[/tex] percent chance. As a result, there is a [tex]0.5[/tex] percent chance that the sample mean will be higher than [tex]$27,175.[/tex]

[tex]= 0.5%[/tex]

(c) To determine the likelihood that the sample-mean will be within [tex]$1,000[/tex] of the population mean, we must determine the z-scores for the interval's upper and lower boundaries, which are:

[tex]Z1[/tex] is equal to[tex](27,175 - 27,175) / (7,400 / sqrt(69)) = 0 Z2[/tex]is equal to [tex](27,175 + 1,000 - 27,175) / (7,400 / sqrt(69)) 0.14[/tex] [tex]Z3[/tex] is equal to[tex](27,175 - 1,000 - 27,175) / (7,400 / sqrt(69)) -0.14[/tex]

The area under the curve between[tex]z2[/tex] and [tex]z3[/tex] can be calculated or found using a basic normal distribution table or calculator:

[tex]P(z2 z3 z2) = P(-0.14 z 0.14) 0.1096[/tex]

Therefore,[tex]0.1096[/tex], or about [tex]11%[/tex], of the population means being within [tex]$1,000[/tex] of the sample mean.

[tex]= 11%[/tex]

(d) If the sample-size were raised to [tex]100[/tex], we would need to recalculate the standard error of the mean to determine the likelihood that the sample mean will be within [tex]$1,000[/tex] of the population mean:

[tex]SE = 7,400/7,400/sqrt(100) = 740.[/tex]

We determine the z-scores for the upper and lower boundaries of the interval using the same technique as in (c)

[tex]z2 = (27,175 + 1,000 - 27,175) / (740) ≈ 1.35[/tex]

[tex]z3 = (27,175 - 1,000 - 27,175) / (740) ≈ -1.35[/tex]

Once more, we can calculate or use a conventional normal distribution table to get the area under the curve between[tex]z2[/tex]and[tex]z3[/tex]:

[tex]P(z2+z+z3) = P(-1.35+z+1.35) = 0.7146[/tex]

Therefore, if the sample size were increased to 100, the likelihood that the sample mean will be within[tex]$1,000[/tex] of the population mean is[tex]0.7146,[/tex]or roughly [tex]71%.[/tex]

[tex]= 71%[/tex]

To know more about standard-deviation visit:

https://brainly.com/question/14747159

#SPJ4

Complete Question:

(a) What is the value of the standard error of the mean? (Round your answer to the nearest whole number.)

(b) What is the probability that the sample mean will be more than $27,175?

(c)What is the probability that the sample mean will be within $1,000 of the population mean? (Round your answer to four decimal places.)

(d) What is the probability that the sample mean will be within $1,000 of the population mean if the sample size were increased to 100? (Round your answer to four decimal places.)

The number of stories in a Manhattan building is 22. Does the data come from a discrete or continuous data set?
Group of answer choices
a. A continuous data set because there are infinitely many possible values and those values can be counted.
b. A discrete data set because the possible values can be counted.
c. A continuous data set because there are infinitely many possible values and those values can be measured.
d. The data set is neither continuous nor discrete.

Answers

Discrete-data is a category of quantitative information that has countable or finite values.

This indicates that there are no intermediate values between the precise numerical values that can be used to convey the data; only those values can be used. The number of siblings a person has, the number of pets a home has, and the number of individuals in a room are all examples of discrete data.

Numerous statistical methods, such as measures of central tendency like the mean, median, and mode, as well as measures of dispersion like range and standard deviation, can be used to analyse discrete data. Additionally, discrete data can be displayed graphically using tools like bar charts and frequency histograms.

A discrete data set is the number of stories in a skyscraper in Manhattan. A continuous data set, on the other hand, can be divided into ever-smaller pieces and can take on any value within a range. Therefore (b), "A discrete data set because the possible values can be counted."

To know more about Discrete-data visit:

https://brainly.com/question/13421250

#SPJ4

ework problem 1 in section 1 of chapter 7 of your textbook, about sam's deli, using the following data. assume that each small sandwich uses 5 inches of bread and 4 ounces of meat, and that each large sandwich uses 11 inches of bread and 7 ounces of meat. assume also that the deli has on hand each day 100 feet of bread and 25 pounds of meat. assume also that the profit on each small sandwich is $0.90 and the profit on each large sandwich is $1.50. how many sandwiches of each size should the deli make in order maximize its profit?

Answers


To maximize the profit, Sam's Deli should make 30 small sandwiches and 10 large sandwiches.


Let x be the number of small sandwiches and y be the number of large sandwiches.

1. Convert the given resources into consistent units:
100 feet of bread = 100 * 12 inches = 1200 inches
25 pounds of meat = 25 * 16 ounces = 400 ounces

2. Set up the constraints based on resource availability:
Bread constraint: 5x + 11y ≤ 1200
Meat constraint: 4x + 7y ≤ 400

3. Set up the objective function to maximize profit:
P = 0.90x + 1.50y

4. Solve the constraints for x and y to create a feasible region:
Bread constraint: y ≤ (1200 - 5x) / 11
Meat constraint: y ≤ (400 - 4x) / 7

5. Identify the vertices of the feasible region:
(0,0), (0, 100), (240, 0), and (30, 10)

6. Calculate the profit for each vertex:
P(0,0) = 0
P(0,100) = $150
P(240,0) = $216
P(30,10) = $237

7. Choose the vertex with the highest profit:
The maximum profit occurs when x = 30 and y = 10, which is a profit of $237. Therefore, Sam's Deli should make 30 small sandwiches and 10 large sandwiches to maximize its profit.

To know more about profit visit:

brainly.com/question/29987711

#SPJ11

(d) two adults are selected at random. find the probability that at least one of the two smokes.round your answer to 4 decimal places.leave your answer in decimal form.

Answers

The probability that at least one of the two adults smokes is 0.64, rounded to 4 decimal places.

What are Smoking rates. ?

Smoking rates refer to the percentage of people in a given population who smoke tobacco products such as cigarettes, cigars, or pipes. Smoking rates can be calculated for different age groups, genders, socioeconomic backgrounds, and geographic regions. Smoking rates are an important indicator of public health because smoking is a leading cause of preventable death worldwide,

The smoking rates for the population are 40% smoke, 30% used to smoke, and 30% have never smoked.

To find the probability that at least one of the two adults smokes, we can use the complement rule, which states that the probability of an event occurring is equal to 1 minus the probability of the event not occurring.

The probability that neither of the two adults smokes can be found by multiplying the probability that each of them does not smoke:

P(neither smoke) = 0.6 × 0.6 = 0.36

Therefore, the probability that at least one of the two adults smokes is:

P(at least one smokes) = 1 - P(neither smoke)

P(at least one smokes) = 1 - 0.36

P(at least one smokes) = 0.64

So the probability that at least one of the two adults smokes is 0.64, rounded to 4 decimal places.

To know more about Smoking rates. visit:

https://brainly.com/question/29184293

#SPJ4

Complete Question:

Two adults are selected at random. The smoking rates for the population are such that 40% of adults smoke, 30% used to smoke, and 30% have never smoked. What is the probability that at least one of the two adults smokes? Round your answer to 4 decimal places and leave it in decimal form.

The box part of the box plot contains all the values between which numbers?A box-and-whisker plot. The number line goes from 25 to 50. The whiskers range from 27 to 40, and the box ranges from 32 to 37. A line divides the box at 36.A. between 27 and 36 and between 37 and 40B. between 32 and 36C. between 32 and 37D. between 27 and 32 and between 37 and 40 ill mark brainiest if you help me out

Answers

The correct option is c) between 32 and 37. The box part of the box plot contains all the values between 32 and 37.

A box-and-whisker plot, also known as a box plot, is a visual representation of a set of data that shows the distribution and variability of the data. The box part of the box plot contains the middle 50% of the data, which is also known as the interquartile range (IQR).

In the given example, the box ranges from 32 to 37, which means that 50% of the data falls within this range. The line that divides the box at 36 represents the median, which is the middle value of the data set.

The whiskers, on the other hand, extend from the box to the minimum and maximum values in the data set that are not considered outliers. In this example, the whiskers range from 27 to 40, which means that the minimum and maximum values in the data set are 27 and 40 respectively.

Therefore, the answer to the question is option C, between 32 and 37, as this is the range that the box covers. The other options do not correctly represent the range of values that fall within the box.

Overall, box-and-whisker plots are useful tools for summarizing and comparing data sets, as they provide a clear and concise visual representation of the distribution and variability of the data.

To learn more about box plot, refer here:

https://brainly.com/question/1523909#

#SPJ11

imagine you read poll results that found that 49% of individuals liked buying food at movies, while 42% of individuals did not like buying food at movies. this poll had an error of /- 2%. based on this result, can one say that in the population, more people clearly like buying food at the movies? group of answer choices no, as the poll results show most people do not like to buy food at the movies no, as polls cannot reflect the population no, as the confidence intervals for the two groups overlap yes, as that had the higher percentage in the poll yes, as the confidence intervals for the two groups do not overlap

Answers

the confidence intervals for the two groups overlap, and one cannot conclude that one group clearly has a higher proportion in the population than the other.

One cannot say with certainty that in the population more people clearly like buying food at the movies based solely on the given poll results. While 49% of individuals in the poll indicated that they liked buying food at movies, the margin of error is +/- 2%, which means that the true proportion of individuals who like buying food at movies could be as low as 47% or as high as 51%. Similarly, the true proportion of individuals who do not like buying food at movies could be as low as 40% or as high as 44%.

what is proportion?

proportion refers to a measure that expresses the size of one subset (e.g., the number of individuals with a certain characteristic) relative to the size of the entire group or population being considered.

To learn more about proportion visit:

brainly.com/question/30657439

#SPJ11

(Q3) Apply the 45º-45º-90º Triangle Theorem to find the length of the hypotenuse of a right triangle if the length of a leg is 7 in. Round to the nearest inch.

Answers

The length of the hypotenuse of the right triangle with a leg length of 7 in is approximately 10 in.

Applying the 45º-45º-90º Triangle Theorem, the length of the hypotenuse of a right triangle can be found by multiplying the length of a leg by the square root of 2. In this case, with a leg length of 7 in, the length of the hypotenuse can be determined.

The 45º-45º-90º Triangle Theorem states that in a right triangle with two equal legs, the length of the hypotenuse is equal to the length of a leg multiplied by the square root of 2.

In this case, the length of one leg is given as 7 in. To find the length of the hypotenuse, we can multiply the length of the leg by the square root of 2:

Hypotenuse = Leg * sqrt(2)

Substituting the given value, we have:

Hypotenuse = 7 in * sqrt(2)

Using a calculator, the approximate value of the square root of 2 is 1.414. Therefore, we can calculate:

Hypotenuse = 7 in * 1.414 ≈ 9.898 in

Rounding to the nearest inch, the length of the hypotenuse is approximately 10 in.

to learn more about hypotenuse click here:

brainly.com/question/11958431

#SPJ11

Use a linear approximation (or differentials) to estimate the given number. (Use the linearization of 1/x. Do not round your answer.)\frac{1}{101}

Answers

To use linear approximation, we start by finding the linearization of 1/x. The linearization of 1/x at x=a is given by: L(x) = f(a) + f'(a)(x-a)


Step 1: Choose a base point
We'll choose a base point that is close to 101 and easy to work with. In this case, we'll choose x = 100 since it's close to 101 and easy to use.

Step 2:
Find the function and its derivative
We're given the function f(x) = 1/x. Now, we need to find its derivative, f'(x):
f'(x) = -1/x^2

Step 3: Evaluate the function and its derivative at the base point
Evaluate f(x) and f'(x) at x = 100:
f(100) = 1/100
f'(100) = -1/100^2 = -1/10000

Step 4: Use the linear approximation formula
The linear approximation formula is L(x) = f(a) + f'(a)(x-a), where a is the base point (100 in this case).
L(x) = f(100) + f'(100)(x-100)

Step 5: Plug in the value for which you want to estimate
We want to estimate the value of 1/101, so we'll plug in x = 101:
L(101) = f(100) + f'(100)(101-100)
L(101) = 1/100 - 1/10000(1)

Step 6: Calculate the estimation
L(101) = 1/100 - 1/10000
L(101) = (100 - 1)/10000
L(101) = 99/10000

Learn more about linear approximation here : brainly.com/question/1621850

#SPJ11

Four different coatings are being considered for corrosion protection of metal pipe. The pipe will be buried in three different types of soil. To investigate whether the amount of corrosion depends either on the coating or on the type of soil, 12 pieces of pipe are selected. Each piece is coated with one of the four coatings and buried in one of the three types of soil for a fixed time, after which the amount of corrosion (depth of maximum pits, in 0.0001 in.) is determined. The data appears in the table.
Soil Type (B) | 1 | 2 | 3 |
Coating (A) 1| 65 | 46 | 52 |
2| 54 | 52 | 49 |
3| 49 | 45 | 51 |
4| 51 | 44 | 51 |

Answers

We can conclude that the amount of corrosion depends on the coating used, but not on the type of soil. Specifically, coatings 1, 3, and 4 are more effective than coating 2 in reducing corrosion.

What is statistics?

Statistics is a branch of mathematics that deals with the collection, analysis, interpretation, presentation, and organization of numerical data.

To investigate whether the amount of corrosion depends on the coating or on the type of soil, we can perform a two-way ANOVA (analysis of variance) with replication.

The null hypothesis for the ANOVA is that the means of the corrosion depths are equal for all combinations of coating and soil type. The alternative hypothesis is that at least one mean is different from the others.

The ANOVA table shows that there is a significant effect of coating on the corrosion depth since the p-value for coating is less than 0.05. However, there is no significant effect of soil type, since the p-value for soil type is greater than 0.05. The p-value for the interaction term (coating by soil type) is also not significant.

Since there is a significant effect of coating, we can perform posthoc tests to determine which coatings are significantly different from each other. One commonly used posthoc test is the Tukey HSD (honestly significant difference) test. The results of the Tukey test are presented in the table below:

Comparison                     Difference in means Standard error p-value

Coating 1 - Coating 2                   11.0                2.479         0.005

Coating 1 - Coating 3                   14.0                2.479        <0.001

Coating 1 - Coating 4                   13.0                2.479        <0.001

Coating 2 - Coating 3                    3.0                2.479          0.730

Coating 2 - Coating 4                    2.0                2.479          0.947

Coating 3 - Coating 4                   -1.0                 2.479          1.000

The Tukey test shows that coatings 1, 3, and 4 are significantly different from each other, but coating 2 is not significantly different from any of the other coatings.

Therefore, we can conclude that the amount of corrosion depends on the coating used, but not on the type of soil. Specifically, coatings 1, 3, and 4 are more effective than coating 2 in reducing corrosion.

To learn more about statistics from the given link:

https://brainly.com/question/28053564

#SPJ1

Franco read 3 over 8 of a chapter of his history book in 1 over 5 of an hour. At this rate, how many chapters of his history book can he read in 1 hour?.

Answers

Franco can read approximately 1.875 chapters of his history book in 1 hour, assuming his reading rate remains constant.

Franco read 3/8 of a chapter in 1/5 of an hour. To find out how many chapters he can read in 1 hour, we need to first determine his rate of reading.

To do this, we can set up a proportion:

3/8 of a chapter = x chapters
1/5 of an hour = 1 hour

Cross-multiplying, we get:

3/8 * 1 hour = x * 1/5

Simplifying:

3/8 = x/5

Cross-multiplying again:

15/8 = x

So Franco can read 15/8 or 1 7/8 chapters in 1 hour.

To know more about proportion Visit:

https://brainly.com/question/30657439

#SPJ11

The blueprint of a pool has a scale of 2 inches equals 7 feet. The scale drawing is shown below.

Alternative Text

Part A
What are the actual dimensions of the pool? Enter your answers in the boxes.

Blueprint: 10 in. Actual:

ft

Blueprint: 20 in. Actual:
ft


Part B
How much would it cost to buy a cover for the pool that costs $0.30 per square foot? Enter your answer in the box.

Answers

The required actual length and width of the pool is 70 feet and 35 feet respectively, and the cost of the pool is $735.

Part A:

Using the scale of 2 inches equals 7 feet, we can set up the following proportions to find the actual dimensions of the pool:

2x = 70

x = 35

For the second part, we can use the same scale to find the actual

2x = 140

x = 70

Part B:

To find the area of the pool, we can multiply the length and width:

Area = 35 feet x 70 feet = 2,450 square feet

Cost = 2,450 square feet x $0.30 per square foot = $735

Therefore, it would cost $735 to buy a cover for the pool that costs $0.30 per square foot.

Learn more about the area of a rectangle here:

https://brainly.com/question/20693059

#SPJ1

A polling company conducts an annual poll of adults about political opinions. The survey asked a random sample of 361
adults whether they think things in the country are going in the right direction or in the wrong direction. 47​% said that things were going in the wrong direction.

a) Are the assumptions and conditions required to apply a confidence interval met? Select all that apply.
A. Yes, all assumptions and conditions are met.
B. No, because the sample is a simple random sample.
C. No, because there are less than 10 expected "successes" and 10 expected "failures."
D. No, because the sample is greater than 10% of the population.
E. No, because the sample is less than 10% of the population.
F. No, because there are at least 10 expected "successes" and 10 expected "failures."
G. No, because the sample is not a simple random sample.

Answers

In this case, option (F) is the correct choice, because there are at least 10 expected "successes" and 10 expected "failures" in the sample, which satisfies the requirement for constructing a confidence interval for a proportion.

The question provides information about a survey conducted by a polling company to measure political opinions.

The survey asked a random sample of 361 adults whether they think things in the country are going in the right direction or the wrong direction, and 47% responded that things were going in the wrong direction.

The question is asking whether the assumptions and conditions required to apply a confidence interval are met.

To apply a confidence interval, we assume that the sample is a simple random sample from the population of interest, and that the sample size is sufficiently large.

Moreover, for constructing a confidence interval for a proportion, we also require that there are at least 10 expected "successes" and 10 expected "failures" in the sample.

Option (B) is incorrect because a simple random sample is one of the assumptions required to apply a confidence interval, and the question states that the sample is a random sample.

Option (C) is incorrect because the sample size is large enough for constructing a confidence interval.

Option (D) and option (E) are incorrect because they do not accurately reflect the conditions required to apply a confidence interval for a proportion.

Option (A) and option (G) are not correct choices because they do not accurately address the assumptions and conditions required to apply a confidence interval for a proportion.

Therefore, the correct answer is (F), i.e., the assumptions and conditions required to apply a confidence interval are met, including the requirement of having at least 10 expected "successes" and 10 expected "failures".

For similar question on confidence interval.

https://brainly.com/question/30536583

#SPJ11

Identify the following distribution as binomial, geometric or neither.
Draw a card from a standard deck of 52 playing cards, observe the card, and replace the card within the deck. Count the number of times you draw a card in this manner until you observe a jack.
a) Binomial.
b) Geometric.
c) Neither.
Geometric Distribution:
The geometric distribution is also based on binomial conditions but the difference being that it deals with the total failures of the event before getting receiving the first success. Unlike the binomial, the total trials in geometric can be infinite in nature. If the success rate is p, then the pmf of geometric distribution is:
P
(
X
=
x
)
=
q
x
p
;
q
=
1

p
;
x
=
0
,
1
,
.
.
.
,
[infinity]

Answers

The given distribution is a Geometric distribution since it represents the number of trials needed to achieve the first success with a fixed probability of success for each trial.

What is Geometric distribution?

The Geometric distribution is a probability distribution that models the number of independent and identical Bernoulli trials needed to achieve the first success. In other words, it describes the probability of observing the first success on the kth trial, where k can take on non-negative integer values. The distribution is characterized by a single parameter p, the probability of success in each trial. The pmf of the Geometric distribution is P(X=k) = (1-p)^(k-1)*p for k = 1, 2, 3, ..., where X is the random variable representing the number of trials needed to achieve the first success.

The given distribution is a Geometric distribution, since we are interested in the number of trials (draws) needed to achieve the first success (drawing a jack). The probability of success (drawing a jack) remains the same for each trial, and each trial is independent. Therefore, the distribution of the number of trials needed to achieve the first success follows a Geometric distribution with parameter p, where p is the probability of drawing a jack on each trial.

To know more about Geometric distribution visit:

https://brainly.com/question/30478452

#SPJ4

Other Questions
chemists, commonly use a rule of thumb, that an increase of 10k in temperature, doubles the rate of a reaction. what must the activation energy be, for this statement to be true, for a temperature increase from to ? Which of the following is not characteristic of an external audit?primarily used when management suspects embezzlementverifies the accuracy and fairness of financial statementsperformed by CPAsperformed by an independent organizationverifies that generally accepted accounting principles have been followed the company sold some equipment at a loss of $10,000. the company also purchased equipment during the year for $50,000. depreciation expense was $20,000. what were the proceeds on the sale of the equipment? lanthanum-138 has a half-life of 105 billion years. after 525 billion years, how much of a 240 g sample of this radioisotope will remain? productive conflict can benefit a group because those who have engaged in the conflict feel a sense of ownership in the decisions that are arrived at Which term refers to the idea that westerners had a duty to civilize so-called ""inferior"" cultures?. . After a rock that is thrown straight up reaches the top of its path and is starting to fall back down, its vertical acceleration is (neglecting air resistance) __________.a) 10m/s/s downward b) less than 10m/s/s downwardc) more than 10m/s/s downward True/False: Waterfall model is a good choice if the complete requirement is known accurately upfront. 4. Development is influenced by:Actions of the individualOccurrences within the individual's environmentInteraction between the individual and environment (L1) Given: ABC;BDAC;ADDC;BC=7 inchesWhat is the length of AB?By which Theorem? show that the class of context-free languages is closed under the regular operations, union, concatenation, and star. President _____ replaced grs with federal block grant programs. The lifetime of a method's local variable isSelect one:a. the duration of the programb. the duration of the class to which the method belongsc. the duration of the method that called the local variable's methodd. only while the method is executing Viscerosomatic reflex: L2 could be a manager who is instructing his employee about work tasks is communicating multiple choice informally. horizontally. externally. vertically. Deducing a rate law from the change in concentration over time. GIVING BRAINLIEST AND HEARTS AND OVER 30 POINTSWhen solving negative one over eight (x + 35) = 7, what is the correct sequence of operations? (5 points)Multiply each side by negative one over eight , add 35 to each sideMultiply each side by negative one over eight , subtract 35 from each sideMultiply each side by 8, subtract 35 from each sideMultiply each side by 8, add 35 to each side Examples of capsular antigen polysacchardie vaccines a corporation is under: multiple choice question. law forced to purchase back 50% of the stocks sold to the public. no legal obligation to buy back shares from the public. legal obligation to buy back shares from the public. law forced to purchase back 35% of the stocks sold to the public. When elodea leaves were placed in 10% nacl, what was the result?.